Linear transformation from $R^3$ to $R^2$.












1












$begingroup$


Find the matrix of the linear transformation $Tcolon {Bbb R}^3 to {Bbb R}^2$ such that



$T(1,1,1) = (1,1)$, $T(1,2,3) = (1,2)$, $T(1,2,4) = (1,4)$.



So far, I have only dealt with transformations in the same R. Any help?










share|cite|improve this question











$endgroup$












  • $begingroup$
    The idea remains the same once you fix a basis of $mathbb{R}^3$ and $mathbb{R}^2$. The only difference you'll see is that earlier you would be getting square matrices. Here you'll get a $2times 3$ matrix. Thus in general $T:mathbb{R}^nto mathbb{R}^m$ yields a $m times n$ matrix. Can you see why?
    $endgroup$
    – Swapnil Tripathi
    Mar 29 '17 at 21:17


















1












$begingroup$


Find the matrix of the linear transformation $Tcolon {Bbb R}^3 to {Bbb R}^2$ such that



$T(1,1,1) = (1,1)$, $T(1,2,3) = (1,2)$, $T(1,2,4) = (1,4)$.



So far, I have only dealt with transformations in the same R. Any help?










share|cite|improve this question











$endgroup$












  • $begingroup$
    The idea remains the same once you fix a basis of $mathbb{R}^3$ and $mathbb{R}^2$. The only difference you'll see is that earlier you would be getting square matrices. Here you'll get a $2times 3$ matrix. Thus in general $T:mathbb{R}^nto mathbb{R}^m$ yields a $m times n$ matrix. Can you see why?
    $endgroup$
    – Swapnil Tripathi
    Mar 29 '17 at 21:17
















1












1








1





$begingroup$


Find the matrix of the linear transformation $Tcolon {Bbb R}^3 to {Bbb R}^2$ such that



$T(1,1,1) = (1,1)$, $T(1,2,3) = (1,2)$, $T(1,2,4) = (1,4)$.



So far, I have only dealt with transformations in the same R. Any help?










share|cite|improve this question











$endgroup$




Find the matrix of the linear transformation $Tcolon {Bbb R}^3 to {Bbb R}^2$ such that



$T(1,1,1) = (1,1)$, $T(1,2,3) = (1,2)$, $T(1,2,4) = (1,4)$.



So far, I have only dealt with transformations in the same R. Any help?







linear-algebra matrices linear-transformations






share|cite|improve this question















share|cite|improve this question













share|cite|improve this question




share|cite|improve this question








edited Jan 1 at 20:52









A.Γ.

22.6k32656




22.6k32656










asked Mar 29 '17 at 21:08









AmaCAmaC

14410




14410












  • $begingroup$
    The idea remains the same once you fix a basis of $mathbb{R}^3$ and $mathbb{R}^2$. The only difference you'll see is that earlier you would be getting square matrices. Here you'll get a $2times 3$ matrix. Thus in general $T:mathbb{R}^nto mathbb{R}^m$ yields a $m times n$ matrix. Can you see why?
    $endgroup$
    – Swapnil Tripathi
    Mar 29 '17 at 21:17




















  • $begingroup$
    The idea remains the same once you fix a basis of $mathbb{R}^3$ and $mathbb{R}^2$. The only difference you'll see is that earlier you would be getting square matrices. Here you'll get a $2times 3$ matrix. Thus in general $T:mathbb{R}^nto mathbb{R}^m$ yields a $m times n$ matrix. Can you see why?
    $endgroup$
    – Swapnil Tripathi
    Mar 29 '17 at 21:17


















$begingroup$
The idea remains the same once you fix a basis of $mathbb{R}^3$ and $mathbb{R}^2$. The only difference you'll see is that earlier you would be getting square matrices. Here you'll get a $2times 3$ matrix. Thus in general $T:mathbb{R}^nto mathbb{R}^m$ yields a $m times n$ matrix. Can you see why?
$endgroup$
– Swapnil Tripathi
Mar 29 '17 at 21:17






$begingroup$
The idea remains the same once you fix a basis of $mathbb{R}^3$ and $mathbb{R}^2$. The only difference you'll see is that earlier you would be getting square matrices. Here you'll get a $2times 3$ matrix. Thus in general $T:mathbb{R}^nto mathbb{R}^m$ yields a $m times n$ matrix. Can you see why?
$endgroup$
– Swapnil Tripathi
Mar 29 '17 at 21:17












3 Answers
3






active

oldest

votes


















1












$begingroup$

Group your 3 constraints into a single one:



$$tag{1}T.underbrace{begin{pmatrix}1&1&1\1&2&2\1&3&4end{pmatrix}}_{M}=underbrace{begin{pmatrix}1&1&1\1&2&4end{pmatrix}}_{N}$$



(where the point means matrix product).



(1) is equivalent to $T=N.M^{-1},$ which is a $2 times 3$ matrix.



Up to you for the last calculations.



You should find $begin{pmatrix}1& 0&0\2&-3&2end{pmatrix}.$






share|cite|improve this answer











$endgroup$













  • $begingroup$
    Why compute $M^{-1}$ ? We can simply use elementary linear combinaisons for get $T(e_3)$, and then it is easy to conclude.
    $endgroup$
    – user171326
    Mar 29 '17 at 21:26










  • $begingroup$
    "Because it gives the simplest explanation" (and the easiest way to give the computation to be done to a computer), I would say.
    $endgroup$
    – Jean Marie
    Mar 29 '17 at 21:32










  • $begingroup$
    I have added the result one must find.
    $endgroup$
    – Jean Marie
    Mar 29 '17 at 22:53



















0












$begingroup$

Hint : $T(0,0,1) = (0,2)$ so the last column of the matrix is $begin{pmatrix} 0 \ 2 end{pmatrix}$.



Do you see how to find it ? Do you see how to continue ?






share|cite|improve this answer









$endgroup$





















    0












    $begingroup$

    Hint:
    Try finding $a$, $b$, $c$, $d$, $e$, $f$ such that:
    $$
    left(
    begin{array}{ccc}
    a & b & c \
    d & e & f \
    end{array}
    right)
    left(
    begin{array}{c}
    1\
    1\
    1
    end{array}
    right)
    =
    left(
    begin{array}{c}
    1\
    1
    end{array}
    right)
    $$
    What other matrix equations can you form?






    share|cite|improve this answer









    $endgroup$













      Your Answer





      StackExchange.ifUsing("editor", function () {
      return StackExchange.using("mathjaxEditing", function () {
      StackExchange.MarkdownEditor.creationCallbacks.add(function (editor, postfix) {
      StackExchange.mathjaxEditing.prepareWmdForMathJax(editor, postfix, [["$", "$"], ["\\(","\\)"]]);
      });
      });
      }, "mathjax-editing");

      StackExchange.ready(function() {
      var channelOptions = {
      tags: "".split(" "),
      id: "69"
      };
      initTagRenderer("".split(" "), "".split(" "), channelOptions);

      StackExchange.using("externalEditor", function() {
      // Have to fire editor after snippets, if snippets enabled
      if (StackExchange.settings.snippets.snippetsEnabled) {
      StackExchange.using("snippets", function() {
      createEditor();
      });
      }
      else {
      createEditor();
      }
      });

      function createEditor() {
      StackExchange.prepareEditor({
      heartbeatType: 'answer',
      autoActivateHeartbeat: false,
      convertImagesToLinks: true,
      noModals: true,
      showLowRepImageUploadWarning: true,
      reputationToPostImages: 10,
      bindNavPrevention: true,
      postfix: "",
      imageUploader: {
      brandingHtml: "Powered by u003ca class="icon-imgur-white" href="https://imgur.com/"u003eu003c/au003e",
      contentPolicyHtml: "User contributions licensed under u003ca href="https://creativecommons.org/licenses/by-sa/3.0/"u003ecc by-sa 3.0 with attribution requiredu003c/au003e u003ca href="https://stackoverflow.com/legal/content-policy"u003e(content policy)u003c/au003e",
      allowUrls: true
      },
      noCode: true, onDemand: true,
      discardSelector: ".discard-answer"
      ,immediatelyShowMarkdownHelp:true
      });


      }
      });














      draft saved

      draft discarded


















      StackExchange.ready(
      function () {
      StackExchange.openid.initPostLogin('.new-post-login', 'https%3a%2f%2fmath.stackexchange.com%2fquestions%2f2209317%2flinear-transformation-from-r3-to-r2%23new-answer', 'question_page');
      }
      );

      Post as a guest















      Required, but never shown

























      3 Answers
      3






      active

      oldest

      votes








      3 Answers
      3






      active

      oldest

      votes









      active

      oldest

      votes






      active

      oldest

      votes









      1












      $begingroup$

      Group your 3 constraints into a single one:



      $$tag{1}T.underbrace{begin{pmatrix}1&1&1\1&2&2\1&3&4end{pmatrix}}_{M}=underbrace{begin{pmatrix}1&1&1\1&2&4end{pmatrix}}_{N}$$



      (where the point means matrix product).



      (1) is equivalent to $T=N.M^{-1},$ which is a $2 times 3$ matrix.



      Up to you for the last calculations.



      You should find $begin{pmatrix}1& 0&0\2&-3&2end{pmatrix}.$






      share|cite|improve this answer











      $endgroup$













      • $begingroup$
        Why compute $M^{-1}$ ? We can simply use elementary linear combinaisons for get $T(e_3)$, and then it is easy to conclude.
        $endgroup$
        – user171326
        Mar 29 '17 at 21:26










      • $begingroup$
        "Because it gives the simplest explanation" (and the easiest way to give the computation to be done to a computer), I would say.
        $endgroup$
        – Jean Marie
        Mar 29 '17 at 21:32










      • $begingroup$
        I have added the result one must find.
        $endgroup$
        – Jean Marie
        Mar 29 '17 at 22:53
















      1












      $begingroup$

      Group your 3 constraints into a single one:



      $$tag{1}T.underbrace{begin{pmatrix}1&1&1\1&2&2\1&3&4end{pmatrix}}_{M}=underbrace{begin{pmatrix}1&1&1\1&2&4end{pmatrix}}_{N}$$



      (where the point means matrix product).



      (1) is equivalent to $T=N.M^{-1},$ which is a $2 times 3$ matrix.



      Up to you for the last calculations.



      You should find $begin{pmatrix}1& 0&0\2&-3&2end{pmatrix}.$






      share|cite|improve this answer











      $endgroup$













      • $begingroup$
        Why compute $M^{-1}$ ? We can simply use elementary linear combinaisons for get $T(e_3)$, and then it is easy to conclude.
        $endgroup$
        – user171326
        Mar 29 '17 at 21:26










      • $begingroup$
        "Because it gives the simplest explanation" (and the easiest way to give the computation to be done to a computer), I would say.
        $endgroup$
        – Jean Marie
        Mar 29 '17 at 21:32










      • $begingroup$
        I have added the result one must find.
        $endgroup$
        – Jean Marie
        Mar 29 '17 at 22:53














      1












      1








      1





      $begingroup$

      Group your 3 constraints into a single one:



      $$tag{1}T.underbrace{begin{pmatrix}1&1&1\1&2&2\1&3&4end{pmatrix}}_{M}=underbrace{begin{pmatrix}1&1&1\1&2&4end{pmatrix}}_{N}$$



      (where the point means matrix product).



      (1) is equivalent to $T=N.M^{-1},$ which is a $2 times 3$ matrix.



      Up to you for the last calculations.



      You should find $begin{pmatrix}1& 0&0\2&-3&2end{pmatrix}.$






      share|cite|improve this answer











      $endgroup$



      Group your 3 constraints into a single one:



      $$tag{1}T.underbrace{begin{pmatrix}1&1&1\1&2&2\1&3&4end{pmatrix}}_{M}=underbrace{begin{pmatrix}1&1&1\1&2&4end{pmatrix}}_{N}$$



      (where the point means matrix product).



      (1) is equivalent to $T=N.M^{-1},$ which is a $2 times 3$ matrix.



      Up to you for the last calculations.



      You should find $begin{pmatrix}1& 0&0\2&-3&2end{pmatrix}.$







      share|cite|improve this answer














      share|cite|improve this answer



      share|cite|improve this answer








      edited Mar 29 '17 at 22:52

























      answered Mar 29 '17 at 21:15









      Jean MarieJean Marie

      28.8k41949




      28.8k41949












      • $begingroup$
        Why compute $M^{-1}$ ? We can simply use elementary linear combinaisons for get $T(e_3)$, and then it is easy to conclude.
        $endgroup$
        – user171326
        Mar 29 '17 at 21:26










      • $begingroup$
        "Because it gives the simplest explanation" (and the easiest way to give the computation to be done to a computer), I would say.
        $endgroup$
        – Jean Marie
        Mar 29 '17 at 21:32










      • $begingroup$
        I have added the result one must find.
        $endgroup$
        – Jean Marie
        Mar 29 '17 at 22:53


















      • $begingroup$
        Why compute $M^{-1}$ ? We can simply use elementary linear combinaisons for get $T(e_3)$, and then it is easy to conclude.
        $endgroup$
        – user171326
        Mar 29 '17 at 21:26










      • $begingroup$
        "Because it gives the simplest explanation" (and the easiest way to give the computation to be done to a computer), I would say.
        $endgroup$
        – Jean Marie
        Mar 29 '17 at 21:32










      • $begingroup$
        I have added the result one must find.
        $endgroup$
        – Jean Marie
        Mar 29 '17 at 22:53
















      $begingroup$
      Why compute $M^{-1}$ ? We can simply use elementary linear combinaisons for get $T(e_3)$, and then it is easy to conclude.
      $endgroup$
      – user171326
      Mar 29 '17 at 21:26




      $begingroup$
      Why compute $M^{-1}$ ? We can simply use elementary linear combinaisons for get $T(e_3)$, and then it is easy to conclude.
      $endgroup$
      – user171326
      Mar 29 '17 at 21:26












      $begingroup$
      "Because it gives the simplest explanation" (and the easiest way to give the computation to be done to a computer), I would say.
      $endgroup$
      – Jean Marie
      Mar 29 '17 at 21:32




      $begingroup$
      "Because it gives the simplest explanation" (and the easiest way to give the computation to be done to a computer), I would say.
      $endgroup$
      – Jean Marie
      Mar 29 '17 at 21:32












      $begingroup$
      I have added the result one must find.
      $endgroup$
      – Jean Marie
      Mar 29 '17 at 22:53




      $begingroup$
      I have added the result one must find.
      $endgroup$
      – Jean Marie
      Mar 29 '17 at 22:53











      0












      $begingroup$

      Hint : $T(0,0,1) = (0,2)$ so the last column of the matrix is $begin{pmatrix} 0 \ 2 end{pmatrix}$.



      Do you see how to find it ? Do you see how to continue ?






      share|cite|improve this answer









      $endgroup$


















        0












        $begingroup$

        Hint : $T(0,0,1) = (0,2)$ so the last column of the matrix is $begin{pmatrix} 0 \ 2 end{pmatrix}$.



        Do you see how to find it ? Do you see how to continue ?






        share|cite|improve this answer









        $endgroup$
















          0












          0








          0





          $begingroup$

          Hint : $T(0,0,1) = (0,2)$ so the last column of the matrix is $begin{pmatrix} 0 \ 2 end{pmatrix}$.



          Do you see how to find it ? Do you see how to continue ?






          share|cite|improve this answer









          $endgroup$



          Hint : $T(0,0,1) = (0,2)$ so the last column of the matrix is $begin{pmatrix} 0 \ 2 end{pmatrix}$.



          Do you see how to find it ? Do you see how to continue ?







          share|cite|improve this answer












          share|cite|improve this answer



          share|cite|improve this answer










          answered Mar 29 '17 at 21:12







          user171326






























              0












              $begingroup$

              Hint:
              Try finding $a$, $b$, $c$, $d$, $e$, $f$ such that:
              $$
              left(
              begin{array}{ccc}
              a & b & c \
              d & e & f \
              end{array}
              right)
              left(
              begin{array}{c}
              1\
              1\
              1
              end{array}
              right)
              =
              left(
              begin{array}{c}
              1\
              1
              end{array}
              right)
              $$
              What other matrix equations can you form?






              share|cite|improve this answer









              $endgroup$


















                0












                $begingroup$

                Hint:
                Try finding $a$, $b$, $c$, $d$, $e$, $f$ such that:
                $$
                left(
                begin{array}{ccc}
                a & b & c \
                d & e & f \
                end{array}
                right)
                left(
                begin{array}{c}
                1\
                1\
                1
                end{array}
                right)
                =
                left(
                begin{array}{c}
                1\
                1
                end{array}
                right)
                $$
                What other matrix equations can you form?






                share|cite|improve this answer









                $endgroup$
















                  0












                  0








                  0





                  $begingroup$

                  Hint:
                  Try finding $a$, $b$, $c$, $d$, $e$, $f$ such that:
                  $$
                  left(
                  begin{array}{ccc}
                  a & b & c \
                  d & e & f \
                  end{array}
                  right)
                  left(
                  begin{array}{c}
                  1\
                  1\
                  1
                  end{array}
                  right)
                  =
                  left(
                  begin{array}{c}
                  1\
                  1
                  end{array}
                  right)
                  $$
                  What other matrix equations can you form?






                  share|cite|improve this answer









                  $endgroup$



                  Hint:
                  Try finding $a$, $b$, $c$, $d$, $e$, $f$ such that:
                  $$
                  left(
                  begin{array}{ccc}
                  a & b & c \
                  d & e & f \
                  end{array}
                  right)
                  left(
                  begin{array}{c}
                  1\
                  1\
                  1
                  end{array}
                  right)
                  =
                  left(
                  begin{array}{c}
                  1\
                  1
                  end{array}
                  right)
                  $$
                  What other matrix equations can you form?







                  share|cite|improve this answer












                  share|cite|improve this answer



                  share|cite|improve this answer










                  answered Mar 29 '17 at 21:54









                  D. BritoD. Brito

                  358111




                  358111






























                      draft saved

                      draft discarded




















































                      Thanks for contributing an answer to Mathematics Stack Exchange!


                      • Please be sure to answer the question. Provide details and share your research!

                      But avoid



                      • Asking for help, clarification, or responding to other answers.

                      • Making statements based on opinion; back them up with references or personal experience.


                      Use MathJax to format equations. MathJax reference.


                      To learn more, see our tips on writing great answers.




                      draft saved


                      draft discarded














                      StackExchange.ready(
                      function () {
                      StackExchange.openid.initPostLogin('.new-post-login', 'https%3a%2f%2fmath.stackexchange.com%2fquestions%2f2209317%2flinear-transformation-from-r3-to-r2%23new-answer', 'question_page');
                      }
                      );

                      Post as a guest















                      Required, but never shown





















































                      Required, but never shown














                      Required, but never shown












                      Required, but never shown







                      Required, but never shown

































                      Required, but never shown














                      Required, but never shown












                      Required, but never shown







                      Required, but never shown







                      Popular posts from this blog

                      Can a sorcerer learn a 5th-level spell early by creating spell slots using the Font of Magic feature?

                      Does disintegrating a polymorphed enemy still kill it after the 2018 errata?

                      A Topological Invariant for $pi_3(U(n))$